AIME 2

Algebra Level 5

Let a , b , c a,b,c be the real solution of the system of equations

{ x 3 x y z = 2 y 3 x y z = 6 z 3 x y z = 20 \large \begin{cases} x^3-xyz=2 \\ y^3-xyz=6 \\ z^3-xyz=20\end{cases}

The greatest possible value of a 3 + b 3 + c 3 = m n a^3+b^3+c^3= \dfrac{m}{n} , where m m and n n are coprime positive integers. What is m + n m+n ?

167 234 158 176 204

This section requires Javascript.
You are seeing this because something didn't load right. We suggest you, (a) try refreshing the page, (b) enabling javascript if it is disabled on your browser and, finally, (c) loading the non-javascript version of this page . We're sorry about the hassle.

2 solutions

Anirudh Sreekumar
Apr 27, 2017

x 3 x y z = 2 y 3 x y z = 6 z 3 x y z = 20 Let, x y z = t x 3 = ( t + 2 ) ( 1 ) y 3 = ( t + 6 ) ( 2 ) z 3 = ( t + 20 ) ( 3 ) Multiplying, ( 1 ) , ( 2 ) and ( 3 ) ( x y z ) 3 = ( t + 2 ) ( t + 6 ) ( t + 20 ) t 3 = ( t + 2 ) ( t + 6 ) ( t + 20 ) ( t = x y z ) t 3 = t 3 + 28 t 2 + 172 t + 240 0 = 28 t 2 + 172 t + 240 0 = 7 t 2 + 43 t + 60 0 = ( 7 t 2 + 28 t ) + ( 15 t + 60 ) 0 = ( 7 t + 15 ) ( t + 4 ) t = 4 , 15 7 Adding, ( 1 ) , ( 2 ) and ( 3 ) x 3 + y 3 + z 3 = 28 + 3 t It is clear that in order to maximize the value of a 3 + b 3 + c 3 we have to maximize t t = 15 7 a 3 + b 3 + c 3 = 28 + 3 ( 15 7 ) = 196 45 7 = 151 7 m + n = 151 + 7 = 158 \begin{aligned}x^3-xyz&=2\\ y^3-xyz&=6\\ z^3-xyz&=20\\\\ \text{Let, }xyz&=t\\ x^3&=(t+2)\hspace{5mm}\color{#3D99F6}\small(1)\\ y^3&=(t+6)\hspace{5mm}\color{#3D99F6}\small(2)\\ z^3&=(t+20)\hspace{3mm}\color{#3D99F6}\small(3)\\\\ \text{Multiplying,}&\color{#3D99F6}(1),(2)\color{#333333}\text{ and }\color{#3D99F6}(3)\\ (xyz)^3&=(t+2)(t+6)(t+20)\\ t^3&=(t+2)(t+6)(t+20)\hspace{5mm}\color{#3D99F6}( t=xyz)\\ t^3&=t^3+28t^2+172t+240\\ 0&=28t^2+172t+240 \\ 0&=7t^2+43t+60\\ 0&=(7t^2+28t)+(15t+60)\\ 0&=(7t+15)(t+4)\\ \implies t&=-4,\dfrac{-15}{7}\\\\ \text{Adding,}&\color{#3D99F6}(1),(2)\color{#333333}\text{ and }\color{#3D99F6}(3)\\ x^3+y^3+z^3&=28+3t\\ \text{It is clear that}&\text{ in order to maximize the value of } a^3+b^3+c^3 \text{ we have to maximize } t\\ \implies t&=\dfrac{-15}{7}\\ a^3+b^3+c^3&=28+3\cdot\left(\dfrac{-15}{7}\right)\\ &=\dfrac{196-45}{7}\\ &=\dfrac{151}{7}\\ \implies m+n&=151+7=\color{#EC7300}\boxed{\color{#333333}158}\end{aligned}

Thank you for the logical and elegant solution.

Hana Wehbi - 4 years, 1 month ago

You’re welcome :)

Anirudh Sreekumar - 4 years, 1 month ago
Guilherme Niedu
Apr 27, 2017

Subtracting the first equation from the second:

y 3 x 3 = 4 \large \displaystyle y^3 - x^3 = 4

x = y 3 4 3 \color{#20A900} \boxed{ \large \displaystyle x = \sqrt[3]{y^3 - 4} }

Subtracting the second equation from the third:

z 3 y 3 = 14 \large \displaystyle z^3 - y^3 = 14

z = y 3 + 14 3 \color{#20A900} \boxed{ \large \displaystyle z = \sqrt[3]{ y^3 + 14} }

Plugging this two equations into the second:

y 3 [ ( y 3 4 ) ( y 3 + 14 ) 3 ] y = 6 \large \displaystyle y^3 - \left [ \sqrt[3]{(y^3 - 4)(y^3 + 14)} \right ] y = 6

y 3 6 = [ y 6 + 10 y 3 56 3 ] y \large \displaystyle y^3 - 6 = \left [ \sqrt[3]{y^6 + 10y^3 -56} \right ] y

Cubing both sides:

y 9 18 y 6 + 108 y 3 216 = y 3 ( y 6 + 10 y 3 56 ) \large \displaystyle y^9 - 18y^6 + 108y^3 - 216 = y^3 \left ( y^6 + 10y^3 -56 \right )

y 9 18 y 6 + 108 y 3 216 = y 9 + 10 y 6 56 y 3 \large \displaystyle y^9 - 18y^6 + 108y^3 - 216 = y^9 + 10y^6 -56y^3

28 y 6 164 y 3 + 216 = 0 \large \displaystyle 28y^6 - 164y^3 + 216 = 0

7 y 6 41 y 3 + 54 = 0 \color{#20A900} \boxed{ \large \displaystyle 7y^6 - 41y^3 + 54 = 0 }

y 3 = 41 ± 4 1 2 4 7 54 14 \large \displaystyle y^3 = \frac{41 \pm \sqrt{41^2 - 4\cdot 7 \ 54}}{14}

( i ) y 3 = 27 7 x 3 = 1 7 z 3 = 125 7 \large \displaystyle (i) \ y^3 = \frac{27}{7} \rightarrow x^3 = - \frac{1}{7} \rightarrow \large \displaystyle z^3 = - \frac{125}{7}

Or:

( i i ) y 3 = 2 x 3 = 2 z 3 = 16 \large \displaystyle (ii) \ y^3 = 2 \rightarrow x^3 = -2 \rightarrow z^3 = 16

( i ) (i) leads to:

a 3 + b 3 + c 3 = 151 7 \boxed{\large \displaystyle a^3+b^3 + c^3 = \frac{151}{7} }

While ( i i ) (ii) leads to:

a 3 + b 3 + c 3 = 16 \boxed{\large \displaystyle a^3+b^3 + c^3 = 16 }

So, the maximum value is:

151 7 \color{#20A900} \boxed{\large \displaystyle \frac{151}{7}}

Then:

m = 151 , n = 7 , m + n = 158 \color{#3D99F6} \large \displaystyle m = 151, n = 7, \boxed{\large \displaystyle m + n = 158}

Thank you for the logical and elegant solution. ‏‎

Hana Wehbi - 4 years, 1 month ago

Log in to reply

My pleasure

Guilherme Niedu - 4 years, 1 month ago

0 pending reports

×

Problem Loading...

Note Loading...

Set Loading...